LSAT and Law School Admissions Forum

Get expert LSAT preparation and law school admissions advice from PowerScore Test Preparation.

 Administrator
PowerScore Staff
  • PowerScore Staff
  • Posts: 8917
  • Joined: Feb 02, 2011
|
#41670
Complete Question Explanation
(The complete setup for this game can be found here: lsat/viewtopic.php?t=4937)

The correct answer choice is (B)

If you answered question #19 correctly, question #20 should make sense. In answer choice (B), if K and P work on Friday, then from the last rule Friday must be P’s first day. However, this would create a violation of the fourth rule because S could not work during the week. Hence, answer choice (B) cannot be true and is correct.
User avatar
 kmclean
  • Posts: 4
  • Joined: Jan 05, 2021
|
#82991
Hi there,

I am just confused as to why C would not be the correct answer. If S works on Monday than that means Paul will never work? is that okay/possible?
User avatar
 KelseyWoods
PowerScore Staff
  • PowerScore Staff
  • Posts: 1079
  • Joined: Jun 26, 2013
|
#83024
Hi kmclean!

If S works on Monday, P can also work on Monday! Look carefully at the wording of that fourth rule: "Seymour does not work on any day before the first day Paul works that week." Notice that rule doesn't say that Paul's first day has to be before any day Seymour's works. All it says is that Seymour does not work on any day before Paul's first day. But that leaves open the possibility that Seymour could work on the same day as Paul's first day! S and P1 can be on the same day, as long as we don't have an S before P1, it's totally fine!

Hope this helps!

Best,
Kelsey
 sblack1998
  • Posts: 10
  • Joined: Feb 05, 2020
|
#84848
This question is confusing to me. I didn't select B because it wasn't clear that P only worked on Friday. I thought he could have worked Monday and Friday without any violation of rule 4 and then answer B could be true, and therefore not the correct answer. Where did my reasoning go wrong?
 Rachael Wilkenfeld
PowerScore Staff
  • PowerScore Staff
  • Posts: 1358
  • Joined: Dec 15, 2011
|
#84883
Hi Slack,

This is a tricky one. The last rule is critical here. Any day K works is the first day that his co-worker works. So if Friday is K and P, that means that P could not have worked before Friday. Because P is with K, it would have to be the first day that P worked. And we know that P couldn't only go on that Friday with K, because then there would be no way that S could work. It's the combination of the two rules that blocks answer choice (B) from occuring.

Hope that helps!

Get the most out of your LSAT Prep Plus subscription.

Analyze and track your performance with our Testing and Analytics Package.